Jump to content

Photo

Đề chọn đội tuyển Quảng Bình 2013-2014

hsg

  • Please log in to reply
20 replies to this topic

#1
namcpnh

namcpnh

    Red Devil

  • Hiệp sỹ
  • 1153 posts

Mình gửi file lên, ai đó bấm ra dùm.

Attached Files


Cùng chung sức làm chuyên đề hay cho diễn đàn tại :

Dãy số-giới hạn, Đa thức , Hình học , Phương trình hàm , PT-HPT-BPT , Số học.

Wolframalpha đây


#2
thukilop

thukilop

    Thượng sĩ

  • Thành viên
  • 291 posts

Nguồn: mathscope.org

Câu 1 (4.0 điểm). 



Giải phương trình $x=\sqrt{3-x}.\sqrt{4-x}+\sqrt{4-x}.\sqrt{5-x}+\sqrt{5-x}.\sqrt{3-x}$
Câu 2 (4.0 điểm). 
Cho a là số thực dương tùy ý. Xét dãy số $({{x}_{n}})$ được xác định như sau:
${{x}_{1}}=a\,;\,\,{{x}_{n+1}}=\frac{{{x}_{n}}\sqr t{2+\sqrt{2+...+\sqrt{2}}}}{{{x}_{n}}+1}$, (tử số có n dấu căn); $\forall n=1,2,3...$
Tính giới hạn của dãy số $({{x}_{n}})$.
Câu 3 (4.0 điểm).
Tìm các hàm số $f:\mathbb{R}\to \mathbb{R}$ thỏa mãn:
$\frac{1}{2}f(xy)+\frac{1}{2}f(xyz)-f(x)f(yz)\ge \frac{1}{4},\forall x,y,z\in \mathbb{R}$.
Câu 4 (4.0 điểm).
Cho tam giác ABC và M, N là hai điểm di động trên đường thẳng BC sao cho $\overrightarrow{MN}=\overrightarrow{BC}$. Đường thẳng d1 đi qua M và vuông góc với AC, đường thẳng d2 đi qua N và vuông góc với AB. Gọi K là giao điểm của d1 và d2. Chứng minh rằng trung điểm I của đoạn AK luôn nằm trên một đường thẳng cố định.
Câu 5 (4.0 điểm).
Chứng minh rằng trong 39 số tự nhiên liên tiếp bất kỳ luôn có ít nhất một số có tổng các chữ số chia hết cho 11.

Vòng II

Bài 6 (5,0 điểm). 
Giải hệ phương trình: $\left\{ \begin{align}
& 9{{y}^{4}}+24{{y}^{3}}-x{{y}^{2}}+7{{y}^{2}}=16-x+24y \\ 
& 8{{y}^{3}}+9{{y}^{2}}+20y-\sqrt[3]{6y+1}+15=x \\ 
\end{align} \right.\,\,\,\,\,\,\,\,\,\,\,\,\,(x,y\in \mathbb{R}\,)$

Bài 7(5,0 điểm). 
Cho các số thực dương x, y, z thỏa mãn: $xyz = 8$. Tìm giá trị nhỏ nhất của:
$P=\frac{1}{{{\left( 1+x \right)}^{3}}}+\frac{8}{{{\left( 2+y \right)}^{3}}}+\frac{64}{{{\left( 4+z \right)}^{3}}}$.
Bài 8(5,0 điểm). 
Cho hai đường tròn (I) và (J) cắt nhau tại A và B sao cho $IA\bot JA$. Đường thẳng IJ cắt hai đường tròn tại C, E, D, F sao cho các điểm C, I, E, D, J, F nằm trên đường thẳng theo thứ tự đó. BE cắt đường tròn (I) tại điểm thứ hai K và cắt AC tại M. BD cắt đường tròn (J) tại điểm thứ hai L và AF tại N.
a) Chứng minh rằng: $MN\bot AB$.
b) Chứng minh rằng: $KE.LN.ID=JE.KM.LD$.
Bài 9(5,0 điểm). 
Cho các số nguyên dương n, k, p với $k\ge 2$ và $k\left( p+1 \right)\le n$. Cho n điểm phân biệt cùng nằm trên một đường thẳng. Tô n điểm đó bằng hai màu xanh, đỏ (mỗi điểm chỉ tô đúng một màu). Tìm số cách tô màu khác nhau, sao cho các điều kiện sau đồng thời được thỏa mãn:
1) Có đúng k điểm được tô bởi màu xanh.
2) Giữa hai điểm màu xanh liên tiếp (tính từ trái qua phải) có ít nhất p điểm được tô màu đỏ.
3) Ở bên phải điểm tô màu xanh cuối cùng có ít nhất p điểm được tô màu đỏ.
(Hai cách tô màu được gọi là khác nhau nếu có ít nhất một điểm được tô màu khác nhau trong hai cách đó).

 

 

http://forum.mathsco...ead.php?t=44695

 


-VƯƠN ĐẾN ƯỚC MƠ-


#3
LNH

LNH

    Bất Thế Tà Vương

  • Hiệp sỹ
  • 581 posts

Câu 1 (4.0 điểm). 

Giải phương trình $x=\sqrt{3-x}.\sqrt{4-x}+\sqrt{4-x}.\sqrt{5-x}+\sqrt{5-x}.\sqrt{3-x}$

Chém câu dễ cái đã :))

Đặt $a=\sqrt{3-x},b=\sqrt{4-x},c=\sqrt{5-x}$

Ta có:

$x=ab+bc+ca$

$a^2+x=3$ $\Leftrightarrow \left ( a+b \right )\left ( a+c \right )=3$

Tương tự, $\left ( b+c \right )\left ( b+a \right )=4$, $\left ( c+a \right )\left ( c+b \right )=5$

Dễ dàng tính được rằng $x=\frac{671}{240}$


Edited by lenhathoang1998, 08-09-2013 - 08:42.


#4
LNH

LNH

    Bất Thế Tà Vương

  • Hiệp sỹ
  • 581 posts

Câu 5 (4.0 điểm).

Chứng minh rằng trong 39 số tự nhiên liên tiếp bất kỳ luôn có ít nhất một số có tổng các chữ số chia hết cho 11.

 

Giả sử các số đó là $a_1<a_2<...<a_39$. Xét 20 số hạng đầu tiên sẽ tồn tại 2 số có chữ số tận cùng là 0 và sẽ có một trong 2 số có chữ số hàng chục khác 9. Gọi số đó là $n$

Xét 19 số $n+1,n+2,...,n+19$, ta có:

$S\left ( n+i \right )=S\left ( n \right )+i$ với $i=1,...,9$

Và $S\left ( n+19 \right )=S\left ( n \right )+10$

Ta có dãy số $S\left ( n \right )+1,...,S\left ( n \right )+10$

Theo nguyên tắc Dirichlet thì một trong các số sau phải chia hết cho 11, suy ra đpcm



#5
LNH

LNH

    Bất Thế Tà Vương

  • Hiệp sỹ
  • 581 posts

Bài 9(5,0 điểm). 

Cho các số nguyên dương n, k, p với $k\ge 2$ và $k\left( p+1 \right)\le n$. Cho n điểm phân biệt cùng nằm trên một đường thẳng. Tô n điểm đó bằng hai màu xanh, đỏ (mỗi điểm chỉ tô đúng một màu). Tìm số cách tô màu khác nhau, sao cho các điều kiện sau đồng thời được thỏa mãn:
1) Có đúng k điểm được tô bởi màu xanh.
2) Giữa hai điểm màu xanh liên tiếp (tính từ trái qua phải) có ít nhất p điểm được tô màu đỏ.
3) Ở bên phải điểm tô màu xanh cuối cùng có ít nhất p điểm được tô màu đỏ.
(Hai cách tô màu được gọi là khác nhau nếu có ít nhất một điểm được tô màu khác nhau trong hai cách đó).

Gọi $x_i$ là khoảng cách giữa điểm tô màu đỏ thứ $i-1$ và $i$

Đặt:

$a_1=x_1$

$a_2=x_2-p$

$a_3=x_3-p$

$a_{k+1}=x_{k+1}-p$

Ta có số cách tô màu thoả mãn yêu cầu đề bài là số nghiệm nguyên không âm của PT:

$a_1+a_2+...+a_{k+1}=n-k\left ( p+1 \right )$

Áp dụng bài toán chia kẹo Euler, ta tính được bằng $C_{n-p\left ( k+1 \right )+k}^{k}$



#6
nhatduy01

nhatduy01

    Trung sĩ

  • Thành viên
  • 132 posts

 

 

Vòng II

Bài 6 (5,0 điểm). 
Giải hệ phương trình: $\left\{ \begin{align}
& 9{{y}^{4}}+24{{y}^{3}}-x{{y}^{2}}+7{{y}^{2}}=16-x+24y \\ 
& 8{{y}^{3}}+9{{y}^{2}}+20y-\sqrt[3]{6y+1}+15=x \\ 
\end{align} \right.\,\,\,\,\,\,\,\,\,\,\,\,\,(x,y\in \mathbb{R}\,)$

 

Phương trình thứ nhất của hệ tương đương

                                 $(y^{2}-1)(9y^{2}+24y+16)=x(y^{2}-1)$

                                 $\Leftrightarrow \begin{bmatrix} y=\pm 1 \Rightarrow x=...& & \\ 9y^{2}+24y+16=x (*)& & \end{bmatrix}$   

Thay (*) vào phương trình thứ hai của hệ ta có

                                                         $8y^{3}-4y-1=\sqrt[3]{6y+1}$

Đặt $2b=\sqrt[3]{6y+1}$.Khi đó ta có hệ

                                                                 $\left\{\begin{matrix} 8b^{3}=6y+1 & & \\ 8y^{3}=2b+4y+1 & & \end{matrix}\right.$

                                                                  $\Rightarrow (b-y)(4b^{2}+4by+4y^{2}+1)=0\Leftrightarrow b=y$

                                                                  $\Rightarrow 8y^{3}=6y+1$

                                                                    ...  



#7
bangbang1412

bangbang1412

    Độc cô cầu bại

  • Phó Quản lý Toán Cao cấp
  • 1670 posts

Bài 7 : Đặt $x=a,y=2b,z=4c=>abc=1$ xong sau đó chứng minh tương tự như ở đây http://diendantoanho...aca3ab3-geq-38/


$$[\Psi_f(\mathbb{1}_{X_{\eta}}) ] = \sum_{\varnothing \neq J} (-1)^{\left|J \right|-1} [\mathrm{M}_{X_{\sigma},c}^{\vee}(\widetilde{D}_J^{\circ} \times_k \mathbf{G}_{m,k}^{\left|J \right|-1})] \in K_0(\mathbf{SH}_{\mathfrak{M},ct}(X_{\sigma})).$$


#8
Juliel

Juliel

    Thượng úy

  • Thành viên
  • 1240 posts

 

Bài 7(5,0 điểm). 
Cho các số thực dương x, y, z thỏa mãn: $xyz = 8$. Tìm giá trị nhỏ nhất của:
$P=\frac{1}{{{\left( 1+x \right)}^{3}}}+\frac{8}{{{\left( 2+y \right)}^{3}}}+\frac{64}{{{\left( 4+z \right)}^{3}}}$.

 

Bổ đề : Nếu $a,b,c,d>0$ và $abcd=1$ thì $\frac{1}{(1+a)^{2}}+\frac{1}{(1+b)^{2}}+\frac{1}{(1+c)^{2}}+\frac{1}{(1+d)^{2}}\geq 1$ (China TST 2004)

Chứng minh bổ đề :

Ta có : $$\frac{1}{(1+x)^{2}}+\frac{1}{(1+y)^{2}}\geq \frac{1}{1+xy}\Leftrightarrow (xy+1)(x-y)^{2}\geq 0$$ đúng với mọi $x,y>0$.

Áp dụng bài toán này, ta có :

$$\frac{1}{(1+a)^{2}}+\frac{1}{(1+b)^{2}}+\frac{1}{(1+c)^{2}}+\frac{1}{(1+d)^{2}}\geq \frac{1}{1+ab}+\frac{1}{1+cd}=\frac{ab+cd+2}{(ab+1)(cd+1)}=\frac{ab+cd+2}{abcd+ab+cd+1}=1$$

Cho $d=1$ ta được : $$\frac{1}{(1+a)^{2}}+\frac{1}{(1+b)^{2}}+\frac{1}{(1+c)^{2}}\geq \frac{3}{4}$$

Quay trở lại bài toán :

Đặt $x=a,y=2b,z=4c\Rightarrow xyz=8abc=8\Rightarrow abc=1$

Khi đó $$P=\left ( \frac{1}{1+x} \right )^{3}+\left ( \frac{2}{2+y} \right )^{3}+\left ( \frac{4}{4+z} \right )^{3}=\left ( \frac{1}{1+a} \right )^{3}+\left ( \frac{1}{1+b} \right )^{3}+\left ( \frac{1}{1+c} \right )^{3}$$

Đặt $\frac{1}{1+a}=A,\frac{1}{1+b}=B,\frac{1}{1+c}=C$ thì $P=A^{3}+B^{3}+C^{3}$ 

Và theo bổ đề ta được $A^{2}+B^{2}+C^{2}\geq \frac{3}{4}$

Áp dụng BĐT $Cauchy-Shwarz$ :

$$P=A^{3}+B^{3}+C^{3}\geq \frac{(A^{2}+B^{2}+C^{2})^{2}}{A+B+C}\geq \frac{(A^{2}+B^{2}+C^{2})^{2}}{\sqrt{3(A^{2}+B^{2}+C^{2})}}=\frac{(A^{2}+B^{2}+C^{2})^{3/2}}{\sqrt{3}}\geq \frac{3}{8}$$

Kết luận : $MinP=\frac{3}{8}\Leftrightarrow a=b=c=1\Leftrightarrow x=\frac{y}{2}=\frac{z}{4}=1\Leftrightarrow x=1,y=2,z=4$


Edited by Juliel, 08-09-2013 - 21:26.

Đừng rời xa tôi vì tôi lỡ yêu người mất rồi !
 

Welcome to My Facebook !


#9
quocbaolqd11

quocbaolqd11

    Hạ sĩ

  • Thành viên
  • 73 posts

Gọi $x_i$ là khoảng cách giữa điểm tô màu đỏ thứ $i-1$ và $i$

Đặt:

$a_1=x_1$

$a_2=x_2-p$

$a_3=x_3-p$

$a_{k+1}=x_{k+1}-p$

Ta có số cách tô màu thoả mãn yêu cầu đề bài là số nghiệm nguyên không âm của PT:

$a_1+a_2+...+a_{k+1}=n-k\left ( p+1 \right )$

Áp dụng bài toán chia kẹo Euler, ta tính được bằng $C_{n-p\left ( k+1 \right )+k}^{k}$

 đáp án sai rồi, bài này là Việt Nam TST năm 1997, bài làm chưa chính xác vì đây là bài toán trên đường tròn, có thể trong TH nào đó điểm tô xanh đầu tiên trong 1 trường hợp khác lại bị trùng với điểm tô xanh thứ $i$ nào đó với $1 \le i \le k$, cách làm của em chỉ phù hợp với bài toán trên đường thẳng vì không xét tới thứ tự điểm. DƯới đây là lời giải.

Attached Files


Edited by quocbaolqd11, 08-09-2013 - 22:03.


#10
LNH

LNH

    Bất Thế Tà Vương

  • Hiệp sỹ
  • 581 posts

 đáp án sai rồi, bài này là Việt Nam TST năm 1997, bài làm chưa chính xác vì đây là bài toán trên đường tròn, có thể trong TH nào đó điểm tô xanh đầu tiên trong 1 trường hợp khác lại bị trùng với điểm tô xanh thứ $i$ nào đó với $1 \le i \le k$, cách làm của em chỉ phù hợp với bài toán trên đường thẳng vì không xét tới thứ tự điểm. DƯới đây là lời giải.

Đề bảo là đường thẳng mà anh?



#11
quocbaolqd11

quocbaolqd11

    Hạ sĩ

  • Thành viên
  • 73 posts

Đề bảo là đường thẳng mà anh?

ồ xin lỗi :luoi:, hoa mắt thật rồi, nhìn nhầm thành "đường tròn". Để đến bù cho cái hoa mắt thì mình xin giải câu dãy số:

bằng quy nạp chứng minh được $\sqrt{2+\sqrt{2+..........+\sqrt{2}}}=2\cos \frac{\pi}{2^{n+1}}$

Thật vậy, $u_1=2\cos \frac{\pi}{2^2}$, giả sử $u_k=2\cos \frac{\pi}{2^{k+1}}$ Ta sẽ chỉ ra rằng $u_{k+1}=\sqrt{2+u_k}$, điều này hiển nhiên đúng, vì
$u_{k+1}=\sqrt{2+u_k}=\sqrt{2 (1+\cos \frac{\pi}{2^{k+1}})}=2.\cos \frac{\pi}{2^{k+2}}$

Đến đây, ta có thể viết lại dãy ban đầu dưới dạng

$x_{n+1}=\frac{2.x_n.\cos \frac{\pi}{2^{k+1}}}{x_n+1}$

Dễ chỉ ra rằng $x_n>0$, biến đổi

$\frac{1}{x_{n+1}}=\frac{1}{2\cos \frac{\pi}{2^{n+1}}}+\frac{1}{2x_n\cos \frac{\pi}{2^{n+1}}}$

Đặt $\frac{1}{x_n}=a_n$ suy ra

$a_{n+1}=\frac{a_n}{2\cos \frac{\pi}{2^{n+1}}}+\frac{1}{2\cos \frac{\pi}{2^{n+1}}}$

Tiếp tục đặt $a_n=b_n.\sin \frac{\pi}{2^n}$, được:

$\frac{\sin \frac{\pi}{2^{n+1}}.b_{n+1}}{4}=\frac{b_n.\sin \frac{\pi}{2^n}}{8\cos \frac{\pi}{2^{n+1}}}+\frac{1}{2\cos \frac{\pi}{2^{n+1}}} \Leftrightarrow b_{n+1}=b_n+\frac{4}{\sin \frac{\pi}{2^n}}$

Đến đây, dễ dàng tìm được công thức của $b_n$ là $b_n=b_1+4\cot \frac{\pi}{2^n}$

Suy ra,

$a_n=\frac{b_1\sin \frac{\pi}{2^n}}{4}+\cos \frac{\pi}{2^n}$
 



#12
nhatduy01

nhatduy01

    Trung sĩ

  • Thành viên
  • 132 posts

 

 

Bài 8(5,0 điểm). 
Cho hai đường tròn (I) và (J) cắt nhau tại A và B sao cho $IA\bot JA$. Đường thẳng IJ cắt hai đường tròn tại C, E, D, F sao cho các điểm C, I, E, D, J, F nằm trên đường thẳng theo thứ tự đó. BE cắt đường tròn (I) tại điểm thứ hai K và cắt AC tại M. BD cắt đường tròn (J) tại điểm thứ hai L và AF tại N.
a) Chứng minh rằng: $MN\bot AB$.
b) Chứng minh rằng: $KE.LN.ID=JE.KM.LD$.

 

 


 

a/Ta có $\angle BAF=\angle BEF,\angle CAB=\angle CDB\Rightarrow \angle EDB+\angle DEB=\angle CAF$

               $\Rightarrow 180^{o}-\angle MBN=\angle MAN\Rightarrow \angle MAN+\angle MBN=180^{o}$

            $\Rightarrow$ Tứ giác AMBN nội tiếp $\angle AMN=\angle ABD=\angle ACD$$\Rightarrow MN$ song song CD $\Rightarrow$ MN vuông góc AB.

b/Do IA vuông góc JA nên IA,JA lần lượt là tiếp tuyến của đường tròn (J),(I) tại A.

                 $\Rightarrow \angle IAE=\angle AFE=\angle EFB=\angle EAB$$\Rightarrow$ AE là phân giác $\angle IAB$

            Có $\angle CAI=\angle ACI=\angle ICB=\angle BAD$

                                          $\Rightarrow \angle CAE=\angle EAD$ hay AE là phân giác góc $\angle CAB$

          Tương tự cũng có BE là phân giác góc $\angle CBA$

               Có $\angle CAK=\angle CBK=\angle KBD$

                   Do tứ giác AMBN nội tiếp nên $\angle MAN+\angle KBD=180^{o}$ hay $\angle CAK+\angle MAN=180^{o}$

            $\Rightarrow$ K,A,F thẳng hàng.

  CMTT ta cũng có AD,BD lần lượt là phân giác $\angle EAD,\angle EBF$ và L,A,C thẳng hàng.

       theo t/c phương tích ta có

                    LN.LB=LA.LM   và   LD.LB=LA.LC   $\Rightarrow$ LN.LC=LD.LM

                    Tương tự   KE.KN=KM.KF

         Gọi O là giao điểm của CF và AB. Ta có theo hệ thức lượng trong tam giác vuông thì $ID^{2}=IA^{2}=IO.IJ,JE^{2}=JA^{2}=JO.IJ$  $\Rightarrow \frac{JE^{2}}{ID^{2}}=\frac{JO}{IO}$(*).

            $\Rightarrow (KE.LN.ID).\frac{ID}{IO}.KN.LC=(JE.KM.LD).\frac{JE}{JO}.KF.LM$
            Với đpcm ta cần chứng minh $\frac{ID}{IO}.KN.LC=\frac{JE}{JO}.KF.LM$

                                                       $\Leftrightarrow \frac{ID.JO.KN.LC}{IO.JE.KF.LM}=1$

                                                       $\Leftrightarrow \frac{MN}{EF}.\frac{CD}{MN}.\frac{ID.JO}{IO.JE}=1$

                                                       $\Leftrightarrow \frac{CD}{EF}.\frac{ID.JO}{IO.JE}=1$

                                                       $\Leftrightarrow \frac{2ID^{2}}{2JE^{2}}.\frac{JO}{IO}=1$

                                                       $\Leftrightarrow \frac{ID^{2}}{JE^{2}}.\frac{JE^{2}}{ID^{2}}=1$(đúng) (do (*))

                 $\Rightarrow$ ĐPCM  



#13
namcpnh

namcpnh

    Red Devil

  • Hiệp sỹ
  • 1153 posts

Gọi $x_i$ là khoảng cách giữa điểm tô màu đỏ thứ $i-1$ và $i$

Đặt:

$a_1=x_1$

$a_2=x_2-p$

$a_3=x_3-p$

$a_{k+1}=x_{k+1}-p$

Ta có số cách tô màu thoả mãn yêu cầu đề bài là số nghiệm nguyên không âm của PT:

$a_1+a_2+...+a_{k+1}=n-k\left ( p+1 \right )$

Áp dụng bài toán chia kẹo Euler, ta tính được bằng $C_{n-p\left ( k+1 \right )+k}^{k}$

 

Bài giải có vấn đề. Đề cho là giữa 2 điểm tô màu đỏ có "ít nhất " $p$ điểm tô màu xanh , nhưng bài giải là giữa 2 điểm tô màu đỏ có đúng $p$ điểm tô màu xanh.


Cùng chung sức làm chuyên đề hay cho diễn đàn tại :

Dãy số-giới hạn, Đa thức , Hình học , Phương trình hàm , PT-HPT-BPT , Số học.

Wolframalpha đây


#14
LNH

LNH

    Bất Thế Tà Vương

  • Hiệp sỹ
  • 581 posts

Bài giải có vấn đề. Đề cho là giữa 2 điểm tô màu đỏ có "ít nhất " $p$ điểm tô màu xanh , nhưng bài giải là giữa 2 điểm tô màu đỏ có đúng $p$ điểm tô màu xanh.

Không phải có đúng $p$ điểm màu xanh đâu anh ạ. Ở đây số điểm màu xanh luôn lớn hơn hoặc bằng $p$ được giải dựa trên tinh thần của bài toán chia kẹo Euler



#15
Tran Phan Kali

Tran Phan Kali

    Lính mới

  • Thành viên
  • 3 posts

Câu 4 (4.0 điểm).
Cho tam giác ABC và M, N là hai điểm di động trên đường thẳng BC sao cho $\overrightarrow{MN} = \overrightarrow{BC}$MN=BC. Đường thẳng d1 đi qua M và vuông góc với AC, đường thẳng d2 đi qua N và vuông góc với AB. Gọi K là giao điểm của d1 và d2. Chứng minh rằng trung điểm I của đoạn AK luôn nằm trên một đường thẳng cố định.

 

*BÀI LÀM:

 

Ta có: $\overrightarrow{MN} = \overrightarrow{BC}$ => $BC \equiv MN$ và BC=MN

Gọi H là trực tâm của $\bigtriangleup ABC$

gọi A' là ảnh của A qua phép tịnh tiến theo $\overrightarrow{BM}$

=> $\bigtriangleup ABC \mapsto \bigtriangleup A'MN$ => $H \mapsto K$ => HK//MN => HK vuông góc với AH => A,H,M,K cùng thuộc đường tròn (I) => I thuộc trung trực AH (cố định)


Edited by Tran Phan Kali, 23-09-2013 - 22:19.


#16
AnnieSally

AnnieSally

    Thiếu úy

  • Thành viên
  • 647 posts
VÒNG $1$
Câu 1. (4.0 điểm). 
Giải phương trình $$x=\sqrt{3-x}.\sqrt{4-x}+\sqrt{4-x}.\sqrt{5-x}+\sqrt{5-x}.\sqrt{3-x}$$
 
Câu 2. (4.0 điểm). 
Cho $a$ là số thực dương tùy ý. Xét dãy số $({{x}_{n}})$ được xác định như sau:
${{x}_{1}}=a\,;\,\,{{x}_{n+1}}=\frac{{{x}_{n}}\sqrt{2+\sqrt{2+...+\sqrt{2}}}}{{{x}_{n}}+1}$, (tử số có $n$ dấu căn); $\forall n=1,2,3...$
Tính giới hạn của dãy số $({{x}_{n}})$.
 
Câu 3. (4.0 điểm).
Tìm các hàm số $f:\mathbb{R}\to \mathbb{R}$ thỏa mãn:
$$\frac{1}{2}f(xy)+\frac{1}{2}f(xyz)-f(x)f(yz)\ge \frac{1}{4},\forall x,y,z\in \mathbb{R}$$.
 
Câu 4. (4.0 điểm).
Cho tam giác $ABC$ và $M,N$ là hai điểm di động trên đường thẳng $BC$ sao cho $\overrightarrow{MN}=\overrightarrow{BC}$. Đường thẳng $d_1$ đi qua $M$ và vuông góc với $AC$, đường thẳng $d_2$ đi qua $N$ và vuông góc với $AB$. Gọi $K$ là giao điểm của $d_1$ và $d_2$. Chứng minh rằng trung điểm $I$ của đoạn $AK$ luôn nằm trên một đường thẳng cố định.
 
Câu 5. (4.0 điểm).
Chứng minh rằng trong $39$ số tự nhiên liên tiếp bất kỳ luôn có ít nhất một số có tổng các chữ số chia hết cho $11$.
 
VÒNG $2$
 
Bài 6. (5,0 điểm). 
Giải hệ phương trình: $$\left\{\begin{matrix} 9y^4+24y^3-xy^2+7y^2=16-x+24y & \\ 8y^3+9y^2+20y-\sqrt[3]{6y+1}+15=x & \end{matrix}\right. (x,y \in \mathbb{R})$$
 
Bài 7. (5,0 điểm). 
Cho các số thực dương $x,y,z$ thỏa mãn: $xyz = 8$. Tìm giá trị nhỏ nhất của:
$$P=\frac{1}{{{\left( 1+x \right)}^{3}}}+\frac{8}{{{\left( 2+y \right)}^{3}}}+\frac{64}{{{\left( 4+z \right)}^{3}}}$$.
 
Bài 8. (5,0 điểm). 
Cho hai đường tròn $(I)$ và $(J)$ cắt nhau tại $A$ và $B$ sao cho $IA\bot JA$. Đường thẳng $IJ$ cắt hai đường tròn tại $C,E,D,F$ sao cho các điểm $C,I,E,D,J,F$ nằm trên đường thẳng theo thứ tự đó. $BE$ cắt đường tròn $(I)$ tại điểm thứ hai $K$ và cắt $AC$ tại $M$. $BD$ cắt đường tròn $(J)$ tại điểm thứ hai $L$ và $AF$ tại $N$.
a) Chứng minh rằng: $MN\bot AB$.
b) Chứng minh rằng: $KE.LN.ID=JE.KM.LD$.
 
Bài 9. (5,0 điểm). 
Cho các số nguyên dương $n,k,p$ với $k\ge 2$ và $k\left( p+1 \right)\le n$. Cho $n$ điểm phân biệt cùng nằm trên một đường thẳng. Tô $n$ điểm đó bằng hai màu xanh, đỏ (mỗi điểm chỉ tô đúng một màu). Tìm số cách tô màu khác nhau, sao cho các điều kiện sau đồng thời được thỏa mãn:
1) Có đúng $k$ điểm được tô bởi màu xanh.
2) Giữa hai điểm màu xanh liên tiếp (tính từ trái qua phải) có ít nhất $p$ điểm được tô màu đỏ.
3) Ở bên phải điểm tô màu xanh cuối cùng có ít nhất $p$ điểm được tô màu đỏ.
(Hai cách tô màu được gọi là khác nhau nếu có ít nhất một điểm được tô màu khác nhau trong hai cách đó).

--- Hết ---


Edited by AnnieSally, 01-11-2013 - 19:40.


#17
kfcchicken98

kfcchicken98

    Thượng sĩ

  • Thành viên
  • 259 posts

sao không ai giải thế

để mình mở hàng :))

bài 7

bđt tương đương: $\frac{1}{(1+x)^{3}}+\frac{1}{(1+\frac{y}{2})^{3}}+\frac{1}{(1+\frac{z}{4})^{3}}$
đặt $\frac{x}{1}=a; \frac{y}{2}=b;\frac{z}{4}=c$, có $abc=1$

có $\frac{1}{(1+a)^{3}}+\frac{1}{(1+a)^{3}}+\frac{1}{8}\geq \frac{3}{2}\frac{1}{(a+1)^{2}}$

tương tư, suy ra $2\sum \frac{1}{(a+1)^{3}}+\frac{3}{8}\geq \frac{3}{2}\sum \frac{1}{(a+1)^{2}}$
giờ sẽ CM $\frac{1}{(a+1)^{2}}+\frac{1}{(b+1)^{2}}+\frac{1}{(c+1)^{2}}\geq \frac{3}{4}$
có $\frac{1}{(a+1)^{2}}+\frac{1}{(b+1)^{2}} \geq \frac{1}{1+ab}$ ( $ab(a-b)^{2}+(ab-1)^{2}\geq 0$)

suy ra VT lớn hơn or bằng $\frac{1}{1+ab}+\frac{1}{(1+c)^{2}}=\frac{c}{c+1}+\frac{1}{(1+c)^{2}}=\frac{c^{2}+c+1}{(c+1)^{2}}$
giả sử $c\geq a;b$, suy ra $c^{3}\geq 1$, suy ra $c\geq 1$

xét đạo hàm của $\frac{c^{2}+c+1}{(c+1)^{2}}$= $\frac{(c^{2}-1)}{(c+1)^{4}}\geq 0$

suy ra hàm số đồng biến, và do $c\geq 1$ nên $f(c)\geq f(1)= \frac{3}{4}$

suy ra 

$2(\frac{1}{(1+a)^{3}}+ \frac{1}{(1+b)^{3}}+\frac{1}{(1+c)^{3}})\geq \frac{9}{8}-\frac{3}{8}=\frac{3}{4}$

suy ra Min= $\frac{3}{8}$ khi x=1; y=2; z=4


Edited by kfcchicken98, 04-11-2013 - 05:18.


#18
nntien

nntien

    Sĩ quan

  • Thành viên
  • 372 posts

Bài 1:

C1:

Nhận thấy $0<x<3$ => $4-x>0$

Chia hai vế cho $4-x$, rồi đặt $a=\frac{1}{4-x}$.

Sau khi biến đổi và rút gon ta được:

$\sqrt{1-a}+\sqrt{1+a}+\sqrt{1-a^2}=4a-1$ => $\frac{1}{4} < a <1$

Đặt $a=sint$ => $cost>0$ta được pt:

$\sqrt{1-sint}+\sqrt{1+sint}=4sint-\sqrt{1-sin^2t}-1$

Bình phương hai vế và rút gọn ta được:

$sint(15sint-8cost-8)=0$

$\Leftrightarrow 15sint-8cost=8$

Giải phương trình trên ta được: 

$sint=\frac{2.8.15}{8^2+15^2}=\frac{240}{289}$

=> $a=\frac{240}{289}$

=> $x=\frac{671}{240}$

C2:

Khi x tăng VT tăng => VP giảm

Khi x giảm VT giảm => VP tăng

=> pt có một nghiệm duy nhất thuộc (0;3)

Nhận thấy $x=\frac{671}{240}$ là nghiệm => đây là nghiệm duy nhất.

Vậy pt đã cho có nghiệm $x=\frac{671}{240}$

P/S: bài này hay!


Edited by nntien, 05-11-2013 - 08:49.

$Maths$$Smart Home$ and $Penjing$

123 Phạm Thị Ngư


#19
namcpnh

namcpnh

    Red Devil

  • Hiệp sỹ
  • 1153 posts

Bài hàm: 

 

Chon $x=y=z=0$ ta có $f(0)=\frac{1}{2}$

 

Chọn $x=y=z=1$ ta có $f(1)=\frac{1}{2}$

 

Chọn $y=0$ ta có $f(x)\leq \frac{1}{2}$

 

Chọn $x=1$ ta có $f(y)\geq \frac{1}{2}$

 

=> $f(x)=\frac{1}{2},\forall x\in \mathbb{R}$


Cùng chung sức làm chuyên đề hay cho diễn đàn tại :

Dãy số-giới hạn, Đa thức , Hình học , Phương trình hàm , PT-HPT-BPT , Số học.

Wolframalpha đây


#20
nhatduy01

nhatduy01

    Trung sĩ

  • Thành viên
  • 132 posts

Đề đã được đăng ở đây!http://diendantoanho...bình-2013-2014/







Also tagged with one or more of these keywords: hsg

1 user(s) are reading this topic

0 members, 1 guests, 0 anonymous users